_Mobin_

New Member
ارسال ها
279
لایک ها
257
امتیاز
0
#61
پاسخ : لیگ علمی پایا !!!

نه اینجوری باید کل وقتتونو سر این سوال میذاشتین چون مثلا به ازای n=8 اون عدد برابره با 43053283 که چک کردن اینکه اوله یا نه کار خیلی وقت گیریه!
حدس بخش‌پذیر بودن بر 13 را شاید می‌شد زد! از طرفی کافی بود اون موبایل ه اینترنت داشته باشه. بعدش وُلفِرَم و ... .:4:
 

حمید آنالیز

Well-Known Member
ارسال ها
1,351
لایک ها
1,322
امتیاز
113
#62
پاسخ : لیگ علمی پایا !!!

کلید پیشنهادی دوم دبیرستان قرار میدم لطفا دوستان هنگام گزاشتن سوال شماره سوالو قرار ندن و اگه میخواهید سوالارو آپ کنم براتون اینجا بزارم
1)2
2)3
3)نمیدونم!
4)3
5)4
6)1
7)3
8)4
9)3
10)5
11)4
12)2
13)3
14)3
15)نمیدونم!
16)4
17)3
18)4
19)1
20)1
21)5
22)شک دارم و نمیدونم!
23)4
24)نمیدونم!
25)5
لطفا بگین اگه لطی بوده و سوالارو اگه میخاین میتونم آپ کنم

---- دو نوشته به هم متصل شده است ----

برای سه و یک فقط اول میشه. راه من اینجوری بود که اگه n بر سه بخشپذیر نباشه اون وقت داریم:
پس در این حالت فقط برای n=1 این عدد اول میشه. اگه n زوج باشه هم که آقا حمید بالاتر تجزیه شو نوشتن پس فقط n=3 می مونه که اون هم به راحتی میشه بررسی کرد که اوله.
2- کدام یک از اعداد زیر از بقیه بزرگتر است؟
1-
2-
3-
4-
5-
اول بدیهی که
از8بتوان 12 بزرگتره بعد با هم توان کردن سه بتوان 36 و اون به نتیجه میرسیم که 3 بتوان 36 بزرگتره بعد....
به نتیجه میرسیم که
از همه بزرگتره یا این روش یا میتونیم با گرفتن نزدیک ترین توان هر عدد به ده اونارو مقایسه کنیم
 

Dadgarnia

New Member
ارسال ها
1,350
لایک ها
1,127
امتیاز
0
#63
پاسخ : لیگ علمی پایا !!!

کلید پیشنهادی دوم دبیرستان قرار میدم لطفا دوستان هنگام گزاشتن سوال شماره سوالو قرار ندن و اگه میخواهید سوالارو آپ کنم براتون اینجا بزارم
1)2
2)3
3)نمیدونم!
4)3
5)4
6)1
7)3
8)4
9)3
10)5
11)4
12)2
13)3
14)3
15)نمیدونم!
16)4
17)3
18)4
19)1
20)1
21)5
22)شک دارم و نمیدونم!
23)4
24)نمیدونم!
25)5
لطفا بگین اگه لطی بوده و سوالارو اگه میخاین میتونم آپ کنم

---- دو نوشته به هم متصل شده است ----


اول بدیهی که
از8بتوان 12 بزرگتره بعد با هم توان کردن سه بتوان 36 و اون به نتیجه میرسیم که 3 بتوان 36 بزرگتره بعد....
به نتیجه میرسیم که
از همه بزرگتره یا این روش یا میتونیم با گرفتن نزدیک ترین توان هر عدد به ده اونارو مقایسه کنیم
نه به نظر من بهتره كه سوالا رو يكي يكي به صورت ماراتن مانند اينجا بذارين البته وقتي سوالاي سال اول تموم شد!
جوابتون درسته.
3- معادله ي
چند دسته جواب در مجموعه ي اعداد طبيعي دارد؟
 

حمید آنالیز

Well-Known Member
ارسال ها
1,351
لایک ها
1,322
امتیاز
113
#64
پاسخ : لیگ علمی پایا !!!

نه به نظر من بهتره كه سوالا رو يكي يكي به صورت ماراتن مانند اينجا بذارين البته وقتي سوالاي سال اول تموم شد!
جوابتون درسته.
3- معادله ي
چند دسته جواب در مجموعه ي اعداد طبيعي دارد؟
ببخشیندا ولی چرا من بی نهایت جواب میارم؟!:192:
 

حمید آنالیز

Well-Known Member
ارسال ها
1,351
لایک ها
1,322
امتیاز
113
#66
پاسخ : لیگ علمی پایا !!!

اشتباه از من بود جواب صفره چون اگه هر دو طرفو به پیما نه 3 بگیریم فغقط یه حالت پیش میاد که
x^2=3k+1,y^2=3k,z=3k+1
پس:

که در حقیقت باید طرف چپ نامسوای اکید بزرگتر از طرف راست باشه !پس به تناقض میرسیم آیا درسته؟

---- دو نوشته به هم متصل شده است ----

نه ببخشین نوشتم صفر او ن فکرم جای دیگه بود جمله اخرم اشتباهه اون تساوی باعث این میشه که بی نهایت جواب داشته باشه
 

Dadgarnia

New Member
ارسال ها
1,350
لایک ها
1,127
امتیاز
0
#67
پاسخ : لیگ علمی پایا !!!

اشتباه از من بود جواب صفره چون اگه هر دو طرفو به پیما نه 3 بگیریم فغقط یه حالت پیش میاد که
x^2=3k+1,y^2=3k,z=3k+1
پس:

که در حقیقت باید طرف چپ نامسوای اکید بزرگتر از طرف راست باشه !پس به تناقض میرسیم آیا درسته؟
نه غلطه! مثلا يه جواب اين سوال
هست پس صفر نمي تونه باشه.

---- دو نوشته به هم متصل شده است ----

نه ببخشین نوشتم صفر او ن فکرم جای دیگه بود جمله اخرم اشتباهه اون تساوی باعث این میشه که بی نهایت جواب داشته باشه
اصلا اون تساوي اشتباهه! z توان چهار داره چه جوري همون
مي مونه؟
 

حمید آنالیز

Well-Known Member
ارسال ها
1,351
لایک ها
1,322
امتیاز
113
#68
پاسخ : لیگ علمی پایا !!!

نه غلطه! مثلا يه جواب اين سوال
هست پس صفر نمي تونه باشه.

---- دو نوشته به هم متصل شده است ----


اصلا اون تساوي اشتباهه! z توان چهار داره چه جوري همون
مي مونه؟
خوب معلومه دیگه اگه توان چهار کنیم بازم باقی ماندش به سه یک نمیشه مگه؟

---- دو نوشته به هم متصل شده است ----

تمام فهمیدم که چه جوب بزرگی دادم!
 

Dadgarnia

New Member
ارسال ها
1,350
لایک ها
1,127
امتیاز
0
#69
پاسخ : لیگ علمی پایا !!!

خوب معلومه دیگه اگه توان چهار کنیم بازم باقی ماندش به سه یک نمیشه مگه؟
يعني شما ميگين ما متغير ها رو اينجوري بذاريم:
ولي اينجوري نميشه گفت كه بي نهايت جواب داره چون هنوز سه تا شرط داريم اول اينكه
مربع كامل باشه دوم اينكه
مربع كامل باشه و سوم اينكه
توان چهارم باشه پس بايد بي نهايت
با اين شرايط پيدا كنيم.
 

حمید آنالیز

Well-Known Member
ارسال ها
1,351
لایک ها
1,322
امتیاز
113
#70
پاسخ : لیگ علمی پایا !!!

فهمیدم اگه مجموعه جوابو به یه عدد توان چهار فرد ضرب کنیم بی نهایت جواب داره:216:

---- دو نوشته به هم متصل شده است ----

چون تکرار میشه
 

Dadgarnia

New Member
ارسال ها
1,350
لایک ها
1,127
امتیاز
0
#71
پاسخ : لیگ علمی پایا !!!

فهمیدم اگه مجموعه جوابو به یه عدد توان چهار فرد ضرب کنیم بی نهایت جواب داره:216:

---- دو نوشته به هم متصل شده است ----

چون تکرار میشه
نه بازم نميشه الان شما اون جواب هايي كه من نوشتمو در 16 ضرب كنين مي بينين كه توي اون معادله صدق نمي كنه.
اصلا سوالو يه جور ديگه مطرح مي كنم:
همه ي جواب هاي معادله ي
را بدست آوريد. :4:
 

حمید آنالیز

Well-Known Member
ارسال ها
1,351
لایک ها
1,322
امتیاز
113
#72
پاسخ : لیگ علمی پایا !!!

نه بازم نميشه الان شما اون جواب هايي كه من نوشتمو در 16 ضرب كنين مي بينين كه توي اون معادله صدق نمي كنه.
اصلا سوالو يه جور ديگه مطرح مي كنم:
همه ي جواب هاي معادله ي
را بدست آوريد. :4:
آقا علیرضا اون جوابا رو گفتم تو یه عدد توان چهر فرد نه زوج پس بدی هیه که خودxبه توان دو انوyهم همینطوره وz به خود عدد ضرب میشه مثلا اگه x,yتو9 یا25 یا49 یا81و...ضرب کنیم اونوخت z به 3 یا 5 یا 7 یا 9 یا.... ضرب میشه و مجموعه جواب های جدید بدست میاد
 

Dadgarnia

New Member
ارسال ها
1,350
لایک ها
1,127
امتیاز
0
#73
پاسخ : لیگ علمی پایا !!!

آقا علیرضا اون جوابا رو گفتم تو یه عدد توان چهر فرد نه زوج پس بدی هیه که خودxبه توان دو انوyهم همینطوره وz به خود عدد ضرب میشه مثلا اگه x,yتو9 یا25 یا49 یا81و...ضرب کنیم اونوخت z به 3 یا 5 یا 7 یا 9 یا.... ضرب میشه و مجموعه جواب های جدید بدست میاد
حالا درست شد ولي فرد يا زوج بودنش تفاوتي نداره. حالا روي سوال جديد فكر كنين يعني تمام جواب هاي معادله رو بدست بيارين.
 

حمید آنالیز

Well-Known Member
ارسال ها
1,351
لایک ها
1,322
امتیاز
113
#74
پاسخ : لیگ علمی پایا !!!

حالا درست شد ولي فرد يا زوج بودنش تفاوتي نداره. حالا روي سوال جديد فكر كنين يعني تمام جواب هاي معادله رو بدست بيارين.
کدوم معادله؟! اگه همونه که اینجا مارتن نظریه اعداد نیست که!:4:دوما شما سوالارو ادامه بدین!
 

Dadgarnia

New Member
ارسال ها
1,350
لایک ها
1,127
امتیاز
0
#75
پاسخ : لیگ علمی پایا !!!

کدوم معادله؟! اگه همونه که اینجا مارتن نظریه اعداد نیست که!:4:دوما شما سوالارو ادامه بدین!
ما براي همين مي گيم كه به صورت ماراتن سوالا رو بذاريم كه يادگيري بره بالا هدف ما كه فقط به دست آوردن جوابا نيست (البته اين روش از امروز شروع شده و شامل ديروز نميشه! :4:) حل اون معادله كه كاري نداره ولي به سوالات ادامه ميديم اگه كسي حلش كرد جوابشو بذاره.
4- كدام يك از اعداد زير نمي تواند
باشد؟
1- 3 , 2- 2 , 3- 5 , 4- 1 , 5- گزينه هاي 1 و 3
اين يكي رو لطفا با اثبات جوابشو بذارين.
 

حمید آنالیز

Well-Known Member
ارسال ها
1,351
لایک ها
1,322
امتیاز
113
#76
پاسخ : لیگ علمی پایا !!!

آريالا یکی از آقایون اینو بحلن دیگه تا ماراتن راه بیفته مثل آقای دادگر نیا و آقای ذولفقاری و آقای امینی!
ما که مثل شما نیستیم من حال حلیدن چنین سوالایی رو ندارم!:33:
 

m-saghaei

New Member
ارسال ها
338
لایک ها
258
امتیاز
0
#77
پاسخ : لیگ علمی پایا !!!

4- كدام يك از اعداد زير نمي تواند
باشد؟
1- 3 , 2- 2 , 3- 5 , 4- 1 , 5- گزينه هاي 1 و 3
اين يكي رو لطفا با اثبات جوابشو بذارين.
جوابش میشه گزینه یک ؟
من اینجوری رفتم:
اول اینکه
میتونه یا 3k یا 3k+1 یا 3k+2 باشه.که تو هیچکدوم از این حالات اگه چک کنید ب.م.م نمیتونه 3 باشه.
واسه ب.م.م پنج هم
میتونه 5k یا 5k+1 یا ... یا 5k+4 باشه.که تو حالات 5k+1 و 5k+2 و 5k+4 ب.م.م نمیتونه 5 باشه.ولی برای دو تا حالت دیگه اگه اعداد 13 و 10 رو بذاریم ب.م.م برابر با 5 میشه.
پس ب.م.م میتونه 5 هم باشه.
و گزینه ها رو اگه نگاه کنیم گزینه یک جوابه.
.
اگه منظورتون اثبات کلی سواله که من نمیدونم! ولی از رو گزینه این میشه.
 

Dadgarnia

New Member
ارسال ها
1,350
لایک ها
1,127
امتیاز
0
#78
پاسخ : لیگ علمی پایا !!!

جوابش میشه گزینه یک ؟
من اینجوری رفتم:
اول اینکه
میتونه یا 3k یا 3k+1 یا 3k+2 باشه.که تو هیچکدوم از این حالات اگه چک کنید ب.م.م نمیتونه 3 باشه.
واسه ب.م.م پنج هم
میتونه 5k یا 5k+1 یا ... یا 5k+4 باشه.که تو حالات 5k+1 و 5k+2 و 5k+4 ب.م.م نمیتونه 5 باشه.ولی برای دو تا حالت دیگه اگه اعداد 13 و 10 رو بذاریم ب.م.م برابر با 5 میشه.
پس ب.م.م میتونه 5 هم باشه.
و گزینه ها رو اگه نگاه کنیم گزینه یک جوابه.
.
اگه منظورتون اثبات کلی سواله که من نمیدونم! ولی از رو گزینه این میشه.
بله درسته. منظورم همينجوري بود.
خب من سوالاي چرت و پرتشو ميذارم تا يه كم سريعتر پيش بريم!
5- حاصل
:)4:) را بدست آوريد.
 

m-saghaei

New Member
ارسال ها
338
لایک ها
258
امتیاز
0
#79
پاسخ : لیگ علمی پایا !!!

بله درسته. منظورم همينجوري بود.
خب من سوالاي چرت و پرتشو ميذارم تا يه كم سريعتر پيش بريم!
5- حاصل
:)4:) را بدست آوريد.
میتونید معمولی تایپش کنید؟ واسه من عکسه نمیاد.بازش هم میکنم اخطار 503 میاد !
 

Dadgarnia

New Member
ارسال ها
1,350
لایک ها
1,127
امتیاز
0
#80
پاسخ : لیگ علمی پایا !!!

میتونید معمولی تایپش کنید؟ واسه من عکسه نمیاد.بازش هم میکنم اخطار 503 میاد !
نه متاسفانه! بنويسم هم فكر نكنم بتونين بفهمين! صبر مي كنم تا اين لاتكس درست بشه!
 
بالا